Difference between revisions of "2023 AMC 10B Problems/Problem 18"

m (Problem)
(redirect)
(Tag: New redirect)
 
Line 1: Line 1:
==Problem 18==
+
#redirect[[2023 AMC 12B Problems/Problem 15]]
 
 
Suppose 𝑎, 𝑏, and 𝑐 are positive integers such that
 
<math>\dfrac{a}{14}+\dfrac{b}{15}=\dfrac{c}{210}</math>.
 
 
 
Which of the following statements are necessarily true?
 
 
 
I. If <math>\gcd(a,14)=1</math> or <math>\gcd(b,15)=1</math> or both, then <math>\gcd(c,210)=1</math>.
 
 
 
II. If <math>\gcd(c,210)=1</math>, then <math>\gcd(a,14)=1</math> or <math>\gcd(b,15)=1</math> or both.
 
 
 
III. <math>\gcd(c,210)=1</math> if and only if <math>\gcd(a,14)=\gcd(b,15)=1</math>.
 
 
 
== Solution 1 (Guess and check + Contrapositive)==
 
[[being revised]]
 
~Technodoggo
 
 
 
==Solution 2==
 
 
 
The equation given in the problem can be written as
 
<cmath>
 
\[
 
15 a + 14 b = c. \hspace{1cm} (1)
 
\]
 
</cmath>
 
 
 
<math>\textbf{First, we prove that Statement I is not correct.}</math>
 
 
 
A counter example is <math>a = 1</math> and <math>b = 3</math>.
 
Thus, <math>{\rm gcd} (c, 210) = 3 \neq 1</math>.
 
 
 
<math>\textbf{Second, we prove that Statement III is correct.}</math>
 
 
 
First, we prove the ``if'' part.
 
 
 
Suppose <math>{\rm gcd}(a , 14) = 1</math> and <math>{\rm gcd}(b, 15) = 1</math>. However, <math>{\rm gcd} (c, 210) \neq 1</math>.
 
 
 
Thus, <math>c</math> must be divisible by at least one factor of 210. W.L.O.G, we assume <math>c</math> is divisible by 2.
 
 
 
Modulo 2 on Equation (1), we get that <math>2 | a</math>.
 
This is a contradiction with the condition that <math>{\rm gcd}(a , 14) = 1</math>.
 
Therefore, the ``if'' part in Statement III is correct.
 
 
 
Second, we prove the ``only if'' part.
 
 
 
Suppose <math>{\rm gcd} (c, 210) \neq 1</math>. Because <math>210 = 14 \cdot 15</math>, there must be one factor of 14 or 15 that divides <math>c</math>.
 
W.L.O.G, we assume there is a factor <math>q > 1</math> of 14 that divides <math>c</math>.
 
Because <math>{\rm gcd} (14, 15) = 1</math>, we have <math>{\rm gcd} (q, 15) = 1</math>.
 
Modulo <math>q</math> on Equation (1), we have <math>q | a</math>.
 
 
 
Because <math>q | 14</math>, we have <math>{\rm gcd}(a , 14) \geq q > 1</math>.
 
 
 
Analogously, we can prove that <math>{\rm gcd}(b , 15) > 1</math>.
 
 
 
<math>\textbf{Third, we prove that Statement II is correct.}</math>
 
 
 
This is simply a special case of the ``only if'' part of Statement III. So we omit the proof.
 
 
 
All analysis above imply
 
<math>\boxed{\textbf{(E) II and III only}}.</math>
 
 
 
~Steven Chen (Professor Chen Education Palace, www.professorchenedu.com)
 

Latest revision as of 20:45, 15 November 2023